Difference between revisions of "2000 AIME II Problems/Problem 7"
m (→Solution 1.2) |
(→Solution 2) |
||
Line 21: | Line 21: | ||
~peelybonehead | ~peelybonehead | ||
+ | |||
+ | ==Solution 3 (Brute Force)== | ||
+ | |||
+ | Convert each denominator to <math>19!</math> and get the numerators to be <math>9,51,204,612,1428,2652,3978,4862</math> (refer to note). Adding these up we have <math>13796</math> therefore <math>\boxed{137}</math> is the desired answer. | ||
+ | |||
+ | Note: | ||
+ | Notice that each numerator is increased each time by a factor of <math>\frac{17}{3}, frac{16}{4}, \frac{15}{5}, \frac{14}{6},</math> etc. If you were taking the test under normal time conditions, it shouldn't be too hard to bash out all of the numbers but it is priority to be careful. | ||
+ | |||
+ | ~SirAppel | ||
== See also == | == See also == |
Revision as of 11:08, 5 April 2024
Problem
Given that
find the greatest integer that is less than .
Solution
Multiplying both sides by yields:
Recall the Combinatorial Identity . Since , it follows that .
Thus, .
So, and .
Solution 2
Let Applying the binomial theorem gives us Since After some fairly easy bashing, we get as the answer.
~peelybonehead
Solution 3 (Brute Force)
Convert each denominator to and get the numerators to be (refer to note). Adding these up we have therefore is the desired answer.
Note: Notice that each numerator is increased each time by a factor of etc. If you were taking the test under normal time conditions, it shouldn't be too hard to bash out all of the numbers but it is priority to be careful.
~SirAppel
See also
2000 AIME II (Problems • Answer Key • Resources) | ||
Preceded by Problem 6 |
Followed by Problem 8 | |
1 • 2 • 3 • 4 • 5 • 6 • 7 • 8 • 9 • 10 • 11 • 12 • 13 • 14 • 15 | ||
All AIME Problems and Solutions |
The problems on this page are copyrighted by the Mathematical Association of America's American Mathematics Competitions.